1. A discrete time linear shift-invariant system has an impulse response h[n] with h[0] = 1, h[1] = - 1, h[2] - 2, and zero otherwise. The system is given an input sequence x[n] with x[0] - x[2] - 1, and zero otherwise. The number of nonzero samples in the output sequence y[n], and the value of y[2] are, respectively





Ask Your Doubts Here

Type in
(Press Ctrl+g to toggle between English and the chosen language)

Comments

  • By: guest on 02 Jun 2017 12.54 am
    Use convolution to get the result.
Show Similar Question And Answers
QA->A statement followed by two assumptions I and II is given. You have to consider the statement to be true even if it seems to be at variance from commonly known facts. You are to decide which of the given assumptions can definitely be drawn from the given statement. Indicate which one of the four given alternatives is correct ? Statement : If more encouragement is given to Sports, Indians will win more gold medals at the Olympic Games. Assumptions : I. Indians do not win gold medals. II. More enc....
QA->പ്രകാശത്തിൻറെ Red Shift, Blue Shift എന്നീ പ്രതിഭാസങ്ങൾക്ക് കാരണം....
QA->Present ages of Kiran and Syam are in the ratio of 5:4 respectively. Three years hence, the ratio of their ages will become 11:9 respectively. What is Syam"s present age in years?....
QA->Present ages of Kiran and Syam are in the ratio of 5 : 4 respectively. Three years hence, the ratio of their ages will become 11 : 9 respectively. What is Syam"s present age in years?....
QA->What minimum number of non-zero non-collinear vectors is required to produce a zero vector?....
MCQ->A discrete time linear shift-invariant system has an impulse response h[n] with h[0] = 1, h[1] = - 1, h[2] - 2, and zero otherwise. The system is given an input sequence x[n] with x[0] - x[2] - 1, and zero otherwise. The number of nonzero samples in the output sequence y[n], and the value of y[2] are, respectively....
MCQ->A linear time-invariant, causal continuous time system has a rational transfer function with simple poles at s = - 2 and s = - 4, and one simple zero at s = - 1. A unit step u(t) is applied at the input of the system. At steady state, the output has constant value of 1. The impulse response of this system is....
MCQ->The response of a linea, time invariant discrete time system to a unit step input ∪(n) is the unit impulse δ(n). The system response to a ramp input n ∪(n) would be....
MCQ->Consider the following statements: If any root of characteristic equation has a positive real part the impulse response is unbounded and system is unstable.If all the roots of a characteristic equation have negative real parts, the impulse response decays to zero.If one or more non-repeated roots of characteristic equation are on jω axis impulse response is bounded but the system is unstable. Which of the above equations are correct?....
MCQ->The impulse response h[n] of a linear time invariant system is given as If the input to the above system is the sequence ejp n/4, then the output is....
Terms And Service:We do not guarantee the accuracy of available data ..We Provide Information On Public Data.. Please consult an expert before using this data for commercial or personal use | Powered By:Omega Web Solutions
© 2002-2017 Omega Education PVT LTD...Privacy | Terms And Conditions
Question ANSWER With Solution